LSAT and Law School Admissions Forum

Get expert LSAT preparation and law school admissions advice from PowerScore Test Preparation.

User avatar
 Dave Killoran
PowerScore Staff
  • PowerScore Staff
  • Posts: 5852
  • Joined: Mar 25, 2011
|
#60236
Complete Question Explanation
(The complete setup for this game can be found here: lsat/viewtopic.php?t=15932)

The correct answer choice is (E)

The assignment of F to the second meeting has an immediate impact on the choices for the first meeting. We know that only F, R or T could be Garibaldi’s first meeting. With F as the second meeting, the first meeting cannot be with F as that would violate the second rule. The first meeting also cannot be with T as there is no room to place S immediately after T. Consequently, when F is Garibaldi’s second meeting, R must be Garibaldi’s first meeting:

pt44_o04_g1_q3a.png
This powerful deduction immediately eliminates answer choices (A), (B), and (D), each of which contain R. A very insightful test taker might realize at this point which of the foreign dignitaries could be Garibaldi’s fourth meeting, but let’s continue analyzing this problem as if we did not have that insight.

Comparing the remaining two answer choices, (C) and (E), both contain S, so the comparison should focus on M and F, the unique variables in each of the remaining answer choices.

Answer choice (C): A quick test of this answer would be to create a hypothetical that places M into the fourth meeting:

pt44_o04_g1_q3b.png
Placing M fourth immediately forces the two remaining Fs into the fifth and seventh meetings:

pt44_o04_g1_q3c.png
At this point there is no room for the TS block, and so we can eliminate this answer choice since placing M fourth does not allow us to create a viable solution.

Answer choice (E): This is the correct answer choice. The following hypothetical proves that F can be Garibaldi’s fourth meeting:

pt44_o04_g1_q3d.png
You do not have the required permissions to view the files attached to this post.
 Tajadas
  • Posts: 63
  • Joined: Apr 11, 2020
|
#75679
Hello,

Thank you for the explanation. I was able to answer this question myself by using a different process that ultimately took longer than if I followed the process you explained. This is a very common problem for me that I am hoping you can help me with- even though I ultimately get the right answer, I often choose to work with rules that do not get me to the correct answer quickly enough. My process was:

Step 1) I looked for easiest variables to eliminate. I used the TS rule to infer that putting T is slot 4 would force a violation of the F/F rule, so I eliminated T and answer choices B and D.

Step 2) I could not use inference 1) above to eliminate any further answers, so I went back to the drawing board. At this point, I realized that R must be in slot 1. From there, I eliminated A and later C as you describe above.

I see now that my decision to focus on the TS rule was time wasted, as Step 2) would have eliminated B and D anyways. Furthermore, understanding that R must go in Slot 1 made eliminating answer choice C very easy. In all, I spent an unnecessarily long time working with the wrong rule. This is a common issue I have, as it often causes me to start resorting to hypotheticals. While hypotheticals get me to the right answer, they take too long to use so often in the limited amount of time available. Do you have any advice on how to identify which rules are the most important to work with when solving a logic game question?
User avatar
 KelseyWoods
PowerScore Staff
  • PowerScore Staff
  • Posts: 1079
  • Joined: Jun 26, 2013
|
#75719
Hi Tajadas!

It's not so much that you used the wrong rule first, it's that it sounds like you were attacking this question from the wrong direction. The TS rule is still important for making the inference that R must be 1st. But it sounds like you were more focused on what couldn't go 4th, rather than moving forward from the Local restriction and thinking what else do we know if F is 2nd?

With a question like this, I always start by looking to see if I have any Global Not Laws beneath the position in question and then eliminating any answer choices I can based on those Global inferences. Here, I don't have any Not Laws underneath slot 4. So then I move on to creating my mini-diagram using my Local rule. I put F in slot 2 and ask myself, ok, what do I know now that F is in slot 2? What rules are immediately affected by this Local restriction?

In this case, the placement of F immediately effects where the other Fs can go, as well as where S and T can go. If F is in slot 2, I can't have F in 1 or 3 and I can't have T in 1. Combining this with my Global Not Laws (can't have M or S in slot 1), I see that R is the only one left that can be in slot 1, meaning it can't be in 4. I look at my answer choices to see if there's anything that I can eliminate with this new information and it gets rid of (A), (B), and (D). Answer choices (C) and (E) have S in common so I only need to do hypotheticals with M and F to determine which is the correct answer.

So, basically, after you've checked your Global inferences for that position and you're ready to look at your Local restriction, don't start at slot 4 and try to eliminate who can go there. Start with the Local restriction and move forward. What do you know if F is 2nd? Which rules does that immediately effect? Then you keep following that domino chain of inferences, eliminating answer choices as you go, until you're out of things to infer. And if you still have a couple answer choices left after that, you try out your hypotheticals.

Hope this helps!

Best,
Kelsey

Get the most out of your LSAT Prep Plus subscription.

Analyze and track your performance with our Testing and Analytics Package.